237

In honor of April Fools Day $2013$, I'd like this question to collect the best, most convincing fake proofs of impossibilities you have seen.

I've posted one as an answer below. I'm also thinking of a geometric one where the "trick" is that it's very easy to draw the diagram wrong and have two lines intersect in the wrong place (or intersect when they shouldn't). If someone could find and link this, I would appreciate it very much.

Harsh Kumar
  • 2,828
  • 4
  • 20
  • 41
Potato
  • 37,797
  • 15
  • 117
  • 251
  • Are we looking to avoid oldies and elementary *proofs*? – Git Gud Apr 01 '13 at 15:01
  • A grad student I know has a wonderful proof involving Fermat's Last Theorem that $\sqrt2^n$ is irrational for all $n$. – user47805 Apr 01 '13 at 15:06
  • 1
    http://math.stackexchange.com/questions/265853/too-simple-to-be-true/294921#294921 – tom Apr 01 '13 at 15:10
  • @user47805, can you post it, here? –  Apr 01 '13 at 15:24
  • 48
    One of the best *ever* fake, or at least apparently-fake, proofs in mathematics goes more or less as "I've found a truly wonderful proof but the margin is too $\rlap{-----------}{\text{...ejem,ejem...marginal...er...}}$ small to contain it..." – DonAntonio Apr 01 '13 at 15:27
  • Some inspiration for fake proofs: http://school.maths.uwa.edu.au/~berwin/humour/invalid.proofs.html – Seth Apr 01 '13 at 19:32
  • 1
    There is this book http://www.amazon.com/Mathematical-Fallacies-Flaws-Flimflam-Spectrum/dp/0883855291/ref=sr_1_2?ie=UTF8&qid=1364876834&sr=8-2&keywords=mathematical+fallacies with one of the best collection I have seen. – Fixed Point Apr 02 '13 at 04:28
  • See this question http://math.stackexchange.com/questions/2069507/is-every-parallelogram-a-rectangle – Harsh Kumar Mar 07 '17 at 14:17

27 Answers27

189

$$x^2=\underbrace{x+x+\cdots+x}_{(x\text{ times})}$$ $$\frac{d}{dx}x^2=\frac{d}{dx}[\underbrace{x+x+\cdots+x}_{(x\text{ times})}]$$ $$2x=1+1+\cdots+1=x$$ $$2=1$$

Doubt
  • 1,599
  • 11
  • 27
Meow
  • 5,853
  • 6
  • 38
  • 59
  • 49
    Nice one. You took one $x$ as a variable and the other $x$ as constant!. – hrkrshnn Apr 01 '13 at 15:19
  • 47
    Also, the first statement isn't true unless $x$ is a positive integer, so it can hardly be used with differentiation by $x$ regardless. After all, $\sin \pi x=0$ is true for all integers $x$, but that doesn't mean we can differentiate both sides to get equality. @boywholived – Thomas Andrews Apr 01 '13 at 15:26
  • @Thomas Andrews.What you said is right and is a better reason too. – hrkrshnn Apr 01 '13 at 15:37
  • 10
    @boywholived No, your argument is good too - he is essentially differentiating $xy$ by $x$ and then substituting $y=x$. – Thomas Andrews Apr 01 '13 at 15:42
  • $x+x+...+x(\left \lceil x \right \rceil\text{ times})+x\cdot(x-\left \lceil x \right \rceil)$. May be this could be another *fake* example . Proof that $\left \lceil x\right\rceil$ is differentiable!. – hrkrshnn Apr 01 '13 at 15:53
  • @LeonardoHerrera I hope you are trolling ... – iKid Apr 01 '13 at 19:54
  • @LeonardoHerrera "$+$" not "$\times$". – crf Apr 02 '13 at 03:09
  • 1
    http://www.dougshaw.com/findtheerror/FTEdiff.html – spin Apr 02 '13 at 13:26
  • @spin the L'Hopital's one may be worth posting. – Meow Apr 02 '13 at 14:45
  • You would think I was kidding, but I was not. Thank god for the 'delete' button. Go ahead guys, go delete your comment. Go. Now. C'mon, you can do it... – Leonardo Herrera Apr 02 '13 at 17:33
  • @ThomasAndrews, I don't get it - If $\sin \pi x$ is equal to zero, then why isn't its derivative, i.e. $\pi \cos \pi x$ equal to zero too? – HeWhoMustBeNamed Oct 27 '17 at 17:18
  • Should the last line be $x =0$? –  Apr 06 '18 at 19:44
  • rhs is only valid when x is positive integer , and if so then would not be continuous and differentiable – Atul Mishra Oct 18 '18 at 08:50
155

enter image description here

Endless chocolate bar (I do not know the author of this animation)

Artem
  • 13,369
  • 3
  • 32
  • 51
  • 5
    Also see [Missing square puzzle](http://en.wikipedia.org/wiki/Missing_square_puzzle) and [Dissection Fallacy](http://mathworld.wolfram.com/DissectionFallacy.html) (making it into a chocolate bar is pretty recent innovation intended to make the paradox accessible to non-math-enthusiasts that's been going around the memesphere lately). – anon Apr 01 '13 at 18:14
  • 63
    For those wondering... the two large pieces get taller as they move--watch their bottoms closely. – ErikE Apr 01 '13 at 19:25
  • 8
    In light of ErikE's comment, not really math. Still nice eye candy though. Pun intended. – Yoni Rozenshein Apr 01 '13 at 22:41
  • 9
    @Yoni, it is math. See the Expert's link for the missing square puzzle. It's connected to solving $ax+by = 1$ in integers. – KCd Apr 01 '13 at 23:10
  • 3
    @KCd If it was related to the missing square puzzle (as I originally thought), then sure. But it's not. The size of that piece is actually changing through the animation. – Izkata Apr 02 '13 at 14:35
  • 2
    Here is an animation explaining the trick: http://i.imgur.com/nA53dlx.gif – BlueTrin Apr 02 '13 at 15:16
  • Wait, isn't this just the two-dimensional version of Banach-Tarski? – Scott Mutchnik Dec 24 '14 at 16:57
123

Proof that a dog has $9$ legs:

No dog has $5$ legs,

A dog has $4$ more legs than no dog.

A dog has $9$ legs

Source: Foolproof: A Sampling of Mathematical Folk Humour, P Renteln, A Dundes

eem
  • 368
  • 1
  • 3
  • 22
user2153964
  • 1
  • 1
  • 1
  • 2
  • 32
    Cries logical Bobby to Ned, will you dare // A bet, which has most legs, a mare, or no mare. // A mare, to be sure, replied Ned, with a grin, // And fifty I’ll lay, for I’m certain to win. // Quoth Bob, you have lost, sure as you are alive, // A mare has but four legs, and no mare has five. – Pål GD Apr 01 '13 at 17:44
  • 7
    By that theorem, a ham sandwich is better than eternal happiness. – Joe Z. Apr 01 '13 at 19:16
  • 10
    Actually, given that animals are occasionally born with two heads, I think the assumption that no dog has 5 legs is probably invalid. It's probably safer to use a larger number like 100. – Tacroy Apr 01 '13 at 23:03
  • 11
    ^ But what if genetic science finds a way to create a centipedal dog? – Joe Z. Apr 17 '13 at 17:06
  • I'll just leave this here... http://www.youtube.com/watch?v=FavUpD_IjVY – Jesse Smith Jul 31 '13 at 13:35
99

Simple one:

$1 = \sqrt{1} = \sqrt{(-1)\cdot(-1)} = \sqrt{-1}\cdot \sqrt{-1} = i\cdot i = -1$

kyticka
  • 379
  • 3
  • 11
  • 55
    $\sqrt{a\cdot b}=\sqrt{a}\cdot\sqrt{b}$. Is valid only when both $a$ and $b$ are positive. – hrkrshnn Apr 01 '13 at 16:03
  • 110
    @boywholived: yes, the idea is that these are fake proofs. – robjohn Apr 01 '13 at 16:04
  • 36
    A nice variant of this that does not require complex numbers is $-1=(-1)^{1/3}=(-1)^{2/6}=((-1)^2)^{1/6}=1^{1/6}=1$. – Baby Dragon Apr 01 '13 at 16:30
  • 28
    @robjohn Yes...but boywholived is pointing out the problem with the proof...valuable to those who may not see the issue. After all, if the problem with the proof was obvious, it wouldn't qualify as much of a joke. – Beska Apr 01 '13 at 17:37
  • 4
    @boywholived It is valid only when at least one of them is non-negative. – Alraxite Apr 02 '13 at 07:41
  • 1
    It seems that Euler himself was confused with this. See [here]( https://webspace.utexas.edu/aam829/1/m/Euler_files/EulerMonthly.pdf) – Esteban Crespi Apr 02 '13 at 14:39
  • @boywholived You are wrong, it's not valid __only__ when they are both positive. In general $(xy)^z=x^zy^z$ for any __complex__ $y,z$ and real $x\ge0$. – user2345215 Apr 16 '14 at 21:04
  • @BabyDragon although a nice fake proof but: $$b^{\frac{n}{m}}=(b^n)^{\frac{1}{m}}=(b^{\frac{1}{m}})^n$$ requires a sufficient condition that , $b$ is not negative when $n$ is even. – Sufyan Naeem May 08 '15 at 09:55
  • @SufyanNaeem That is exactly what makes it nice, at least in the context of teaching. I have seen way to many textbooks and instructors, either gloss over theses conditions or not even mention them. It is something of a pet peeve of mine. – Baby Dragon May 09 '15 at 22:31
  • @BabyDragon I agree. – Sufyan Naeem May 10 '15 at 09:43
  • 1
    @BabyDragon The cube roots are not needed to show the fallacy. The fallacy can also be presented like this: $-1=(-1)^{2/2}=((-1)^2)^{1/2}=1^{1/2}=1$. – Rosie F May 06 '20 at 07:35
96

Let us prove the following (obviously) wrong proposition:
$\underline{\text{Proposition}}$
All positive natural numbers are definable in under eleven words.

$\underline{\text{Proof}}$
Suppose for the sake of contradiction(!) that not all positive natural numbers are definable in under eleven words. Then there is a smallest integer $n\in\mathbb N$ which is not definable in under eleven words. But this number is $$ \color{brown} { \text { the smallest positive integer not definable in under eleven words, } } $$ therefore, it is definable in ten words. Contradiction!

--

For more information see the wikipedia article Berry paradox.

P..
  • 14,509
  • 2
  • 29
  • 52
  • 15
    So which number is "the largest positive integer definable in ten words or less"? – Joe Z. Apr 01 '13 at 19:17
  • 5
    @JoeZeng "Ackermann with (Ackermann with (Graham, Graham), Ackermann with (Graham, Graham))"? Does that count? – Cole Tobin Apr 02 '13 at 01:02
  • 2
    @JoeZeng why should there need to be one? – crf Apr 02 '13 at 03:12
  • 1
    @Max all nonempty sets of positive natural numbers have a smallest element... – jwg Apr 02 '13 at 09:09
  • @crf: Can you prove there isn't one? – Joe Z. Apr 02 '13 at 12:46
  • @ColeJohnson: I was looking for a proof myself. If you want to play like that, I can do "Tree of tree of tree of tree of Graham's number", where "tree" refers to the [TREE function](http://en.wikipedia.org/wiki/Kruskal's_tree_theorem). – Joe Z. Apr 02 '13 at 12:47
  • Although to be honest, "Tree of tree of tree of tree of eighty-six" would have more than sufficed. – Joe Z. Apr 03 '13 at 12:40
  • 4
    I love that one :) The thing is, in the ZF axioms we have "the seperation schema", which says that if X is a set and phi is a formula, then there is a set Y such that for all a, (a /in Y) iff (a /in X and phi(a)). This is what you use here to get "the set of numbers not definable in 10 words or less". The thing is, the seperation schema demands that phi will be definable in first-order logic, which it isn't here :P – Ekuurh Apr 10 '13 at 18:40
  • 3
    Interestingly, one way to prove that the halting problem is unsolvable is to try to make this rigorous. If you can solve the halting problem, you can write a program that takes an input N, and then finds all outputs of programs of length $M$ with input $K$, with $M,K\leq N$, then find the least natural number not output in any of these cases. Then feed the length of this program to itself. With a solution to the halting problem, you can write this program easily. – Thomas Andrews Dec 17 '14 at 22:48
81

A friend of mine came up with this proof that all perfect numbers are even. A little more advanced than your usual fake proof that $0=1$.

Assume $$2n = \sum_{d|n} d$$

Then by Möbius inversion:

$$n =\sum_{d|n} 2d\cdot\mu\left(\frac{n}{d}\right)$$

and therefore $n$ is even.

Thomas Andrews
  • 164,948
  • 17
  • 196
  • 376
  • 11
    This is a great example! I heard it over 20 years ago, so I doubt your friend is the first person to make this "observation". – KCd Apr 01 '13 at 15:46
  • 8
    @KCd Hah, it was at math camp in the early 80s, so we're talking 20 years ago for me, too. – Thomas Andrews Apr 01 '13 at 15:47
  • 14
    It was at math camp for me too, in the late 80s. Did your camp happen to be located in Ohio? – KCd Apr 01 '13 at 15:52
  • 44
    Yep, the Ross program. :) Guess it was the same instance. – Thomas Andrews Apr 01 '13 at 15:53
  • 36
    @Thomas: You kids! :-) I did it in $1963$, when it was still at Notre Dame. – Brian M. Scott Apr 01 '13 at 20:37
  • 1
    This is pretty cool ... though it would be better if the first perfect number $n=6$ didn't reveal it to be false. – Greg Martin Apr 01 '13 at 22:19
  • @KCd I think Brian was saying he went to the program back then, when it was at Notre Dame, not that this particular proof was from that era. But I could be wrong. – Thomas Andrews Apr 01 '13 at 22:33
  • @Brian, is Tom right about my misunderstanding your comment? – KCd Apr 01 '13 at 22:40
  • 1
    @KCd: Yes, Thomas’s interpretation is correct: I was talking about the program, not the proof. By the way, Wikipedia says that the program started in $1957$, so its longevity is pretty impressive. – Brian M. Scott Apr 01 '13 at 22:48
  • 6
    I know it's not appropriate to "explain" a joke, but is this a valid proof of: "IF _all_ numbers are perfect, THEN all numbers are even." – Jeppe Stig Nielsen Apr 02 '13 at 14:21
  • 5
    @JeppeStigNielsen: Actually a stronger statement: if all divisors of perfect number $n$ are perfect, then $n$ is even. Too bad that $1$ isn't perfect. – Marc van Leeuwen Apr 02 '13 at 15:07
70

$$\int \frac{1}{x}dx = x\cdot\frac{1}{x} - \int x\,d\frac{1}{x}$$ $$\int \frac{1}{x}dx = 1 + \int x\frac{1}{x^2}dx$$ $$\int \frac{1}{x}dx = 1 + \int \frac{1}{x}dx$$ $$0 = 1 $$

Kostya
  • 198
  • 1
  • 9
63

"Proof" that $1$ is the largest natural number.

Let $n$ be the largest natural number. Then, $n^2$, being a natural number, is less than or equal to $n$. Therefore $n^2-n=n(n-1)\leq 0$. Hence, $0\leq n\leq 1$. Therefore $n=1$.

AppliedSide
  • 1,394
  • 13
  • 15
  • 28
    I know these are all joke proofs, but this is also a good reminder about the importance of proving the *existence* of a mathematical object before you go on to prove things about said object. Otherwise you can prove lots of interesting but nonsensical things. – Josh Chen Jul 12 '13 at 01:34
  • 11
    @JoshChen Sometimes it's good to prove some properties of a mathematical object first - if some of them give you an easy proof of nonexistence. – Daniel Fischer Jul 27 '13 at 21:01
  • @JoshChen that's the one I was searching for. Nice objection! – Sufyan Naeem May 08 '15 at 10:00
  • 10
    It _does_ prove that the largest natural number, if it exists, is equal to 1. – gnasher729 Apr 02 '16 at 12:16
  • Do you have any reference about this? I am sure I read it in Wikipedia somewhere but I can't remember it. – drzbir Jan 14 '17 at 00:12
  • 4
    I'm sorry but I cannot seem to find the exact reference, but if I'm not mistaken, I learned of this fake proof from a book by Luc Tartar. His purpose was to illustrate (in the context of the calculus of variations) the danger in assuming the existence of an extremum without proving it. – AppliedSide Jan 18 '17 at 17:40
62

Proof that Riemann hypothesis is true:

(1) At least one of the following statements is true

(2) The above statement is false

(3) Riemann hypothesis is true

Grobber
  • 2,998
  • 1
  • 22
  • 30
  • Interesting, I have a feeling the fallacy may have something to do with Godel's incompleteness theorems, am I close? – Honza Brabec Apr 01 '13 at 17:59
  • 10
    Well I do not know the above theorem, but I know the fallacy is regarding that "Every mathematical statement is not necessarily either true or false". – Grobber Apr 01 '13 at 18:02
  • 1
    For example consider: **"This statement is false"**, the cyclicity generated by its truth and falsehood makes the statement to be classifiable as none. – Grobber Apr 01 '13 at 18:04
  • While I enjoy this sort of self-reference, the real problem with this proof is that it includes unjustified (and potentially false) premises, in the same way that "(1) The following statement is true; (2) The Riemann hypothesis is true" does. – Greg Martin Apr 01 '13 at 22:23
  • 5
    @GregMartin: There is a difference: in your example, both (1) and (2) could be taken to be false, or both to be true, and there is no inconsistency in either case. In the above (fake) proof in the answer though, (1) is not a premise, but can be "proved" to be true: if (1) is false, then both (2) and (3) are false and in particular (2) is false, which means that (1) is true, which is a contradiction. Therefore (1) is true, and therefore (2) is false, and therefore as at least one of (2) and (3) are true, (3) must be true. – ShreevatsaR Apr 02 '13 at 06:28
  • @ShreevatsaR: yes, that is am important difference - but not a difference relevant to my point. I'm pointing out that the flaw in the proof is traceable simply to the first instant, when we accepted the trio-of-statements as being "true" - when in fact there's no reason to think that it is. That is the same as with my more pedestrian example ... and also the same as "Either this sentence is false or the Riemann hypothesis is true". – Greg Martin Apr 02 '13 at 08:29
  • 5
    @GregMartin: I don't understand what you mean by accepting the trio of statements as true: we did no such thing (and in fact "proved" statement (2) to be false). Self-reference is crucial to the argument (the way (1) refers to (2) which refers back to (1)), which is why your first example shows nothing, while your second example with "either this sentence is false..." does. If you mean accepting that each statement in the system is either true or false, yes that is the flaw as Grobber already pointed out above before you first commented — and self-reference is required for it to give anything. – ShreevatsaR Apr 02 '13 at 08:54
  • 5
    @HonzaBrabec: it’s not unrelated to Gödel’s incompleteness; but it’s most closely related, I think, to [Tarski’s undefinability of truth](http://en.wikipedia.org/wiki/Tarski's_undefinability_theorem), which essentially states that no reasonably strong logical system can talk about truth-values of its own statements. The two main ingredients of that are (a) some deep thought to formalize precisely what that self-reference means, and (b) having done this, a quick derivation of a contradiction. Part (b) can be done — among other ways — by the present false proof, with RH replaced by absurdity. – Peter LeFanu Lumsdaine Apr 02 '13 at 14:54
  • @PeterLeFanuLumsdaine: To add a bit to your above comment, not only is it because truth is undefinable in a **classical** theory that interprets arithmetic, it is crucially because of LEM. If one accepts LEM only for statements that can be justified to have a boolean truth value, all such paradoxes (see also Quine's paradox) vanish. Furthermore, it is possible to have self-reflection in a non-classical theory, such as Tarski's theory of truth, but necessarily (by incompleteness) any such theory cannot interpret arithmetic without being uncomputable. – user21820 Apr 21 '19 at 05:29
52

The scientist/engineer's proof that all odd numbers are prime...

  • 1 is prime
  • 3 is prime
  • 5 is prime
  • 7 is prime
  • 9 - experimental error
  • 11 is prime
  • 13 is prime
  • 15 - experimental error
  • 17 is prime
  • 19 is prime

Therefore, by extension (ignoring the scientific errors), all odd numbers are prime.

jwernerny
  • 101
  • 1
  • 4
44

All cats have the same color by induction on the number of cats.

Base case: If there is only one cat, then it doesn't matter the color. They all have the same color.

Inductive step: Fix $n$. Suppose for any set of $n$ cats they have the same color. Consider any set of $n+1$ cats. Then cats 1 through n have the same color by inductive hypothesis. There is only one remaining to check, but we also know that cats $2$ through $n+1$ have the same color by the inductive hypothesis. Thus they must all have the same color (for example, because they all have the same color as the second one in this proof). QED.

...But clearly not all cats have the same color...

apnorton
  • 17,156
  • 4
  • 47
  • 107
Matt
  • 7,010
  • 4
  • 28
  • 44
42

Theorem: If we mark $n$ points on a circle and connect each point to every other point by a straight line, the number of regions that the interior of the circle is divided into is $2^{n-1}$.

Proof: First let's collect numerical evidence.

When $n = 1$ there is one region (the whole disc).

When $n = 2$ there are two regions (two half-discs).

When $n = 3$ there are 4 regions (three lune-like regions and one triangle in the middle).

When $n = 4$ there are 8 regions, and if you're still not convinced then try $n=5$ and you'll find 16 regions if you count carefully.

Our proof in general will be by induction on $n$. Assuming the theorem is true for $n$ points, consider a circle with $n+1$ points on it. Connecting $n$ of them together in pairs produces $2^{n-1}$ regions in the disc, and then connecting the remaining point to all the others will divide the previous regions into two parts, thereby giving us $2 \cdot 2^{n-1} = 2^n$ regions.

Or does it...

KCd
  • 30,396
  • 2
  • 67
  • 110
  • Is this true generically? – Baby Dragon Apr 01 '13 at 16:34
  • 10
    A good lesson in the dangers of both making assumptions based on examples, and of lacking rigor in your proofs. – Jack M Apr 01 '13 at 16:41
  • 1
    The sad part about this particular one for me is that I actually presented this proof in foundations once (set theory, logic). *sigh* – Sean Allred Apr 01 '13 at 16:58
  • "try n=5 and you'll find 16 regions if you count carefully" - it's 11 really, isn't it? – Rotsor Apr 01 '13 at 17:01
  • @Roster: it is 16, not 11. Remember to connect *each* of the 5 points to every other point by a straight line. – KCd Apr 01 '13 at 17:38
  • @Baby Dragon: Try 6 and see what happens. – KCd Apr 01 '13 at 17:39
  • 4
    It depends on the way you place the dots, but the maximum number of regions is given by the sequence $$\{1, 2, 4, 8, 16, 31, 57, 99, 163, 256, 386, 562, 794, 1093, 1471, 1941, 2517, 3214, 4048, 5036, 6196,\\ 7547, 9109, 10903, 12951, 15276, 17902, 20854, 24158, 27841, 31931, 36457, 41449, 46938,\\ 52956, 59536, 66712, 74519, 82993, 92171,\ldots\}$$. i.e. [A006522](http://oeis.org/A006522)$+n$ – Spenser Apr 02 '13 at 14:44
  • 8
    Actually it is in OEIS directly: [A000127](http://oeis.org/A000127) – Spenser Apr 02 '13 at 14:52
  • 6
    It is interesting that there is actually a polynomial expression for this: $$a(n)=\frac{1}{24}n^4-\frac{1}{4}n^3+\frac{23}{24}n^2-\frac{3}{4}n+1$$ – Spenser Apr 02 '13 at 17:23
  • 5
    @Spenser: Yes, the small powers of 2 happen to match the polynomial sequence $1 + \binom{n}{2} + \binom{n}{4}$. Similarly, $1 + \binom{n}{2} + \binom{n}{4} + \cdots + \binom{n}{2r}$ equals $2^{n-1}$ for $1 \leq n \leq 2r+1$, and it equals $2^{n-1} - 1$ for $n = 2(r+1)$. – KCd Apr 02 '13 at 23:00
  • 2
    3Blue1Browm tackled this! https://www.youtube.com/watch?v=84hEmGHw3J8&list=PLZHQObOWTQDOqzmnORfqizZK-TcBE09jR – JavaMan Jul 04 '18 at 02:42
36

Zero$^{th}$ fundamental theorem of calculus: $$\int\limits_{0}^{a}{f(x) dx}=0\space \space \quad \quad \forall a \in \mathbb{R} \text{ and } \forall f(x) \in\mathcal{R}\left(\left(0,a\right)\right)$$


Proof

It's well known that $$\int\limits_{0}^{0}{f(x) dx}=0\space \space \quad \quad \forall f(x) \space \tag{1} $$

Case I

When $a=0$ The theorem at this case follows directly from $(1) $

Case II

When $a\neq 0$

Let $\large I=\int\limits_{0}^{a}{f(x) dx}$ Let $\large \sin{\frac{\pi \cdot x}{a}}=t $. Then $\large \frac{\pi}{a}\cos{\frac{\pi \cdot x}{a}}dx={dt}$ $$\cos{\frac{\pi \cdot x}{a}}=\sqrt{1-\sin^{2}{\frac{\pi \cdot x}{a}}}=\sqrt{1-t^2}$$ $$dx=\frac{a}{\pi}\cdot\frac{dt}{\sqrt{1-t^2}}$$

As we have introduced a substitution we should change the variables of the limit. $$\begin{cases} x=0 & \text{then } t=0, \text{ (lower limit)} \\ x=a & \text{then } t=0, \text{ (upper limit)}\end{cases}$$

We have $$I=\int\limits_{0}^{a}{f(x) dx}=\large \int\limits_{0}^{0}{\left(\frac{f(\frac{a}{\pi}\cdot\arcsin{t})}{\sqrt{1-t^2}} dt\right)}=0$$

Q.E.D.

hrkrshnn
  • 6,061
  • 3
  • 31
  • 60
  • You could change $\pi$ to $a$ , $a\in \mathbb{R}$ and you would get a generalization. – Ishan Banerjee Apr 02 '13 at 09:49
  • 1
    There’s no reason to consider Case I separately — the current Case II covers it anyway (nowhere in Case II do you use the assumption that $a ≠ 0$). – Peter LeFanu Lumsdaine Apr 02 '13 at 14:57
  • 14
    @Peter LeFanu Lumsdaine. I think we should consider case 1 $a=0$. Otherwise our initial substitution $\sin{\frac{\pi \cdot x}{\boxed{a}}}=t$ would . . . .. – hrkrshnn Apr 02 '13 at 15:22
  • I don't get what is wrong with this. Help! – A Googler Apr 03 '15 at 12:39
  • 4
    @AGoogler: The first thing wrong I notice is that when $\dfrac{\pi}{2}<\theta\leq \pi$, $\cos(\theta)\neq \sqrt{1-\sin^2(\theta)}$. – Jonas Meyer Apr 03 '15 at 22:23
  • 1
    @AGoogler The inverse of sine is not well-defined in this domain. – Mathew George May 11 '15 at 06:43
  • 1
    (Sorry for the necropost) But is it also true more generally that because $sin(\frac{\pi x}{a})$ is not a diffeomorphism between domains (it's not invertible), you can't define a proper pullback to integrate the form? – BRSTCohomology Jan 04 '18 at 19:40
33

All numbers are interesting.

Assume there is at least one uninteresting number. Then there is a non-empty set of uninteresting numbers. There is also a smallest uninteresting number. That makes it an interesting number so we remove it from the set of uninteresting numbers and repeat until we have removed all uninteresting numbers from that set.

Keith Wolters
  • 101
  • 1
  • 3
  • 1
    I think you mean to say that the set of uninteresting numbers is non-empty. – Git Gud Apr 01 '13 at 17:18
  • I guess so. Of course if we did start with an empty set of uninteresting numbers there would be no need for the proof. – Keith Wolters Apr 01 '13 at 17:29
  • I mean instead of "then there is a set of uninteresting numbers", so you can then conclude, by the [well ordering principle](http://en.wikipedia.org/wiki/Well-ordering_principle) that "there is also a smallest uninteresting number". – Git Gud Apr 01 '13 at 17:31
  • 16
    I have heard this proof with "There is a smallest uninteresting number. Hey, that's pretty interesting! Contradiction." – Yoni Rozenshein Apr 01 '13 at 18:24
  • Works for the natural numbers at least. I guess it would work for the reals by applying Zorn's lemma... – usul Apr 01 '13 at 18:52
  • [In this blog post](http://blog.plover.com/math/uninteresting-numbers.html) I argue that there exists a minimally interesting real number. – MJD Apr 02 '13 at 06:43
  • @MJD, interestingly (no pun intended), your blog post needs the notion of *interesting property* in order to define *interesting number*. But there are only countably many (describable) properties hene they are readily well-ordered and thus either all or no property is interesting ... – Hagen von Eitzen Apr 02 '13 at 16:53
  • Only if you can prove that there is a canonically interesting well-ordering. Unlike the integers, there is no single pre-eminent ordering of an arbitrary countable set. – MJD Apr 02 '13 at 20:38
  • Is 1729 interesting? – DonielF May 16 '17 at 22:09
26

Here is a another proof that "$0=1"$. $$0=(1-1)+(1-1)+(1-1)+(1-1)+\cdots=1-1+1-1+1\cdots=1+(-1+1)+(-1+1)+(-1+1)+(-1+1)\cdots=1+0+0+\cdots=1$$ Of course, this proof fails because the series $\Sigma (-1)^n$ is divergent.

Baby Dragon
  • 2,723
  • 19
  • 31
  • 10
    How about this one: Let S = 1+2+4+8+16+... Then 2S = 2+4+8+16+32+... Subtraction of first minus second gives -S = 1 ans so S = -1 therefore -1 = 1+2+4+8+16+..... – imranfat Apr 01 '13 at 16:49
  • 13
    @imranfat: That is actually true when working in the $2$-adics... – TMM Apr 01 '13 at 18:59
  • Am I not getting it or we just missed -1 after third equals sign just before the fourth one? – Denys S. Apr 02 '13 at 14:38
  • @TMM: I think proofs like these are canonical examples of proofs which are false in one setting (e.g. standard convergence over the reals), but which in other settings with the same formalism (e.g. p-adics, or formal power series, or even just other notions of convergence/summability over the reals) may turn out to be true, and quite useful. – Peter LeFanu Lumsdaine Apr 02 '13 at 15:08
  • 2
    @Peter The original proof actually has applications in topology as the [Mazur swindle](http://en.wikipedia.org/wiki/Eilenberg–Mazur_swindle) where infinite sums are defined (but addition is not necessarily cancellative). – Erick Wong Apr 02 '13 at 15:25
23

The proof that $\pi = 4$, discussed here: Is value of $\pi = 4$?

  1. Inscribe a unit circle inside a square with side $4$.
  2. Repeat the following: For every vertex of the polygon (at first the square) that does not touch the circle, "fold it" so it touches the circle (by creating two additional vertices, so that all angles are right angles).
  3. The length of the polygon is always $4$, but the polygon converges to the circle, so $\pi = 4$.
Yoni Rozenshein
  • 6,534
  • 22
  • 40
  • 6
    Fractals can be tricky when it comes to perimeter. – Jon Purdy Apr 01 '13 at 18:30
  • 8
    What does converge, however, is the area. – obataku Apr 01 '13 at 22:20
  • 2
    Very true; math is full of such convergent/divergent conundrums. The Gabriel's Horn, for instance, has infinite surface area (because it has infinite length/depth and at every point along its length a nonzero diameter), but a finite convergent volume. – KeithS Apr 01 '13 at 23:08
  • 5
    @KeithS: Careful, if one uses $\frac{1}{x^2}$ to make Gabriel's horn instead of $\frac{1}{x}$, then both the volume and surface area area finite, despite the fact that it "has infinite length/depth and at every point along its length a nonzero diamter." – Jason DeVito Apr 01 '13 at 23:38
  • 6
    So, $\lim_{}{4} =\pi$? – Cole Tobin Apr 02 '13 at 01:17
19

Timothy Chow has posted a proof that $1=0$ via an illicit differentiation under the integral sign on his website, here is a quote:

Proof that $1=0$

Given any $x$, we have (by using the substitution $u=\frac{x^2}{y}$) $$ \int_0^1 \frac{x^3}{y^2}e^{-\frac{x^2}{y}}dy=\left[xe^-{\frac{x^2}{y}}\right]^1_0=xe^{-x^2}. $$ Therefore, for all $x$, $$ \begin{array}{rcl} e^{-x^2}(1-2x^2) &=& \frac{d}{dx} (xe^{-x^2}) \\ &=& \frac{d}{dx} \int_0^1 \frac{x^3}{y^2} e^{-\frac{x^2}{y}} dy \\ &=& \int_0^1 \frac{\partial}{\partial x} \left(\frac{x^3}{y^2} e^{-\frac{x^2}{y}}\right) dy \\ &=& \int_0^1 e^{-\frac{x^2}{y}} \left(\frac{3x^2}{y^2} - \frac{2x^4}{y^3}\right) dy. \end{array} $$ Now set $x=0$; the left-hand side is $e^0(1-0)=1$, but the right-hand side is $\int_0^1 0\ dy=0$.

Potato
  • 37,797
  • 15
  • 117
  • 251
  • 37
    "Illicit differentiation" sounds **so** much more exciting than implicit differentiation. – Adam Saltz Apr 01 '13 at 18:56
  • 5
    There is no need for any kind of differentiation under the integral sign. The essence of the argument is this: for $a > 0$, $\int_1^\infty ae^{-ax}dx = e^{-a}$. Now let $a \rightarrow 0^{+}$. The left side has limit $\int_1^\infty 0dx = 0$ and the right side has limit $e^{-0} = 1$. Maybe not... – KCd Apr 01 '13 at 21:57
19

Here's a "proof" of $e^{x} = 1$ for all $x$. $$\exp(x) = \exp(i2\pi\cdot\frac{x}{i2\pi}) = \exp(i2\pi)^{\frac{x}{i2\pi}} = 1 ^{\frac{x}{i2\pi}} = 1.$$

Long
  • 738
  • 5
  • 10
  • 3
    This is a nice one, would you mind stating which = sign is incorrect? I assume the second one because of incorrect rules of exponents since there are complex numbers involved? – imranfat Apr 01 '13 at 20:09
  • 2
    Yes, it's the second equality :) – Long Apr 01 '13 at 20:13
17

Here's a fake proof that there are no natural numbers $p$ and $q$ such that $\sqrt{2} = \frac{p}{q}$. Suppose it were the case and further suppose, without loss of generality, that $p$ and $q$ are relatively prime. Then $\sqrt{2} = \frac{p}{q}$ and so $$2q^{2} = p^{2}, \tag{*}$$ which means that $p$ is even. Thus $p=2r$ for some natural number $r$. Plugging that into (*) gives $2q^{2} = 4r^{2}$, so that $q^{2} = 2r^{2}$, which means $q$ is also even, contrary to the assumption that $p$ and $q$ are relatively prime.

Quinn Culver
  • 4,217
  • 1
  • 25
  • 45
17

Let $f(x) = 1$. It's easy to see that its Fourier transform is $0$ almost everywhere, so $\hat {\hat f}(x) = 0$. By the inversion theorem, $1 = 0$.

Yoni Rozenshein
  • 6,534
  • 22
  • 40
15

$$\begin{align*} \sum_{k\ge 1}\frac1k&=\sum_{k\ge 0}\left(\frac1{2k+1}+\frac1{2k+2}\right)\\ &=\sum_{k\ge 0}\frac{4k+3}{(2k+1)(2k+2)}\\ &=\sum_{k\ge 0}\left(\frac{2(2k+1)}{(2k+1)(2k+2)}+\frac1{(2k+1)(2k+2)}\right)\\ &=\sum_{k\ge 0}\left(\frac1{k+1}+\frac1{(2k+1)(2k+2)}\right)\\ &=\sum_{k\ge 1}\frac1k+\sum_{k\ge 0}\frac1{(2k+1)(2k+2)}\\ &=\sum_{k\ge 1}\frac1k+\sum_{k\ge 0}\left(\frac1{2k+1}-\frac1{2k+2}\right)\\ &=\sum_{k\ge 1}\frac1k+\sum_{k\ge 1}\frac{(-1)^{k+1}}k\\ &=\sum_{k\ge 1}\frac1k+\ln 2\;, \end{align*}$$

so $\ln 2=0$, and $2=e^{\ln 2}=e^0=1$.


The legitimate version of this shows that $$\sum_{k=1}^{2n}\frac{(-1)^{k+1}}k=H_{2n}-H_n\;,$$ where $H_n$ is the $n$-th harmonic number, and with the observation that

$$\ln\left(2-\frac1{n+1}\right)=\int_{n+1}^{2n+1}\frac{dx}x<H_{2n}-H_n<\frac1{n+1}+\int_n^{2n}\frac{dx}x=\frac1{n+1}+\ln 2$$

is one way to show that $\sum\limits_{k\ge 1}\frac{(-1)^{k+1}}k=\ln 2$.

Brian M. Scott
  • 588,383
  • 52
  • 703
  • 1,170
15

Let me prove that the number $1$ is a multiple of $3$.

To accomplish such a wonderful result we are going to use the symbol $\equiv$ to denote "congruent modulo $3$". Thus, what we need to prove is that $1 \equiv 0$. Next I give you the proof:

$1\equiv 4$ $\quad \Rightarrow \quad$ $2^1\equiv 2^4$ $\quad \Rightarrow \quad$ $2\equiv 16$ $\quad \Rightarrow \quad$ $2\equiv 1$ $\quad \Rightarrow \quad$ $2-1\equiv 1-1$ $\quad \Rightarrow \quad$ $1\equiv 0$.

boumol
  • 1,215
  • 7
  • 16
  • 1
    How "does" $2 \equiv 16 \implies 2 \equiv 1$? Is it because $1 \equiv 4$ and $16 / 4 \in \mathbb{N}$? That doesn't seem to follow... what am I missing? – wchargin Jun 06 '14 at 23:45
  • 3
    @WChargin: That part's just legitimate transitivity, with $16 = 1 \text{ (mod 3)}$. I still can't think of anyone whom the exponentiation right at the start would fool, though. – Vandermonde Oct 23 '15 at 02:27
  • @boumol WHat is the mistake in this argument? – Babai Mar 08 '18 at 18:58
  • 7
    @Babai, $a \equiv b \Rightarrow a^k \equiv b^k$, but does not imply $k^a \equiv k^b$ – Iuri Mielniczuk Jul 25 '18 at 06:39
  • @boumol $2^1 \equiv 2$ mod 3 but $2^4 \equiv 1$ mod 3. Multiplying two natural numbers that are the same mod 3 always gets you a number the same mod 3 but that's not the case for natural number exponentation. – Timothy Jan 03 '19 at 07:03
12

$$(n+1)^2 = n^2+2n+1$$

Expansion: $$(n+1)^2-(2n+1) = n^2$$

Subtract from both sides: $$(n+1)^2-(2n+1)-n(2n+1) = n^2-n(2n+1)$$

Add to both sides: $$(n+1)^2-(n+1)(2n+1) = n^2-n(2n+1)$$

Factor: $$(n+1)^2-(n+1)(2n+1)+\frac{(2n+1)^2}{4} = n^2-n(2n+1)+\frac{(2n+1)^2}{4}$$

Add to both sides: $$\left[(n+1)-\frac{2n+1}{2}\right]^2 = \left[n-\frac{2n+1}{2}\right]^2$$

Square root: $$(n+1)-\frac{2n+1}{2} = n-\frac{2n+1}{2}$$

Subtract from both sides: $$n+1 = n$$

Subtract from both sides: $$1 = 0$$

Impossible!

Civa
  • 103
  • 5
  • 4
    Took me a while to figure this one out. I typed each expression in my TI and left hand side equals right hand side. Until I reache the square root part. The problem is the square root. If A² = B² that does not automatically imply A=B, it could also be A=-B That's an awesome example! – imranfat Apr 02 '13 at 19:48
8

Here's a fun compilation of fake proofs in calculus. One of them is already here, but I'll give another example: \begin{align*} \int_{-1}^1\frac{dx}{1 + x^4} &=\int_{-1}^1\frac{\frac{1}{x^4}}{\frac{1}{x^4} + 1}dx\\ u &= \frac{1}{x^4}\\ du &= -\frac{4}{x^5}dx\\ \int_{-1}^1\frac{dx}{1 + x^4} &=\int_1^1\frac{u}{u+1}\left(-\frac{u^{-5/4}}{4}\right)\,du\\ &= -\frac{1}{4}\int_1^1\frac{du}{\sqrt[4]{u}\left(u + 1\right)}\\ &= 0. \end{align*} However, one look at the graph of $\frac{1}{1 + x^4}$ (or a quick estimate) tells us that the integral in question is certainly nonzero!

Stahl
  • 22,164
  • 4
  • 45
  • 69
  • Please look at the moderator's note at the beginning of the post. – hrkrshnn Apr 01 '13 at 16:47
  • @boywholived Right, I had posted that there because this post had linked there. I have since deleted the answer from the "duplicate" question. – Stahl Apr 01 '13 at 16:48
  • The answer to integral is not incorrect at all! It is just that you wanted to calculate the area under the curve and that didn't work. The answer to an integral itself does not always have to give you the area under the curve unless that is what the question states. – imranfat Apr 01 '13 at 16:49
  • @imranfat It's we can also see that the integral is nonzero because $\frac{1}{1 + x^4} > 0$ on $[-1,1]$. – Stahl Apr 01 '13 at 16:50
  • I agree, so if the question is "Calculate the area under the curve" then you are correct. What I am saying is that the answer to an arbitrary integral does not have to be an area under the curve perse – imranfat Apr 01 '13 at 16:54
  • 1
    @imranfat I agree that in general an integral does not have to represent the area under a curve. However, in this case, we can interpret the integral in question as the area under the curve of the graph of $1/(1 + x^4)$ without fear, and that's one way of seeing that this calculation cannot be correct. – Stahl Apr 01 '13 at 18:18
  • If the calculations are correct and this integral can be interpret as the area under the curve, and the area is not zero, what´s wrong here? – Integral Apr 02 '13 at 14:17
  • 3
    @Integral the substitution we consider is invalid: here's the theorem: Suppose that $f$ is continuous on an open interval $I$. *Let $u$ and $u'$ be continuous on an open interval $J$*, and assume that the range of $u$ is contained in $I$. If $a,b\in J$, then $\int_a^b f(u(x))u'(x)\,dx = \int_{u(a)}^{u(b)} f(u)\,du$. Certainly, the $u$ we use here is not continuous on the interval in question, $(-1 - \epsilon,1 + \epsilon)$. – Stahl Apr 02 '13 at 15:13
  • It's worth remembering that results in calculus can ***always*** be checked by writing a program to calculate successive approximations of the curve/area/arc length etc. –  Aug 05 '15 at 22:59
  • @Stahl can I also say that dividing inside the integral by $x^4$ at the beggining is only valid for $x \neq 0$, but $0$ is inside the integration domain then it is not a valid step? –  Apr 27 '18 at 05:22
6

We shall prove that $x=e^{\pi/2} \approx 4.8$ is a solution to $x^x=x$.

First, we compute $i^i = e^{i \log i} = e^{i(\pi/2i)} = e^{- \pi/2}$. Next we compute $i^{i^i} = e^{-pi/2 i} = -i$. Then $x=i^{i^{i^i}}=(-i)^i=e^{i\log (-i)}=e^{i(-\pi/2 i)}=e^{\pi/2}$.

Finally, $x^i=i^{i^{i^{i^i}}}=e^{i \pi/2}=i$.

Raise each side of the previous equation to the $i^{i^i}$th power. We get $$ x^{i^{i^{i^i}}}={i^{i^{i^i}}}$$ or $$x^x=x$$ where $x=e^{\pi/2}$.

abnry
  • 14,161
  • 2
  • 33
  • 75
2

$1=2$: A Proof using Complex Numbers

This supposed proof uses complex numbers. If you're not familiar with them, there's a brief introduction to them given below. The Fallacious Proof:

Step 1: $-1/1 = 1/-1$
Step 2: Taking the square root of both sides: $\sqrt{-1/1} = \sqrt{1/-1}$.
Step 3: Simplifying: $\sqrt{-1} / \sqrt{1} = \sqrt{1} / \sqrt{-1}$
Step 4: In other words, $i/1 = 1/i$.
Step 5: Therefore, $i / 2 = 1 / (2i)$,
Step 6: $i/2 + 3/(2i) = 1/(2i) + 3/(2i)$,
Step 7: $i (i/2 + 3/(2i) ) = i ( 1/(2i) + 3/(2i) )$,
Step 8: $(i^2)/2 + (3i)/2i = i/(2i) + (3i)/(2i)$,
Step 9: $(-1)/2 + 3/2 = 1/2 + 3/2$,
Step 10: and this shows that $1=2$.

See if you can figure out in which step the fallacy lies.

Ajo Koshy
  • 101
  • 5
2

Whatever goes up, must come down

Rules of Mathematics prove that this is a wrong rule and should be discarded

Proof:

Eq 1. Goes Up = Comes Down

Eq 2. (1 / Goes Up) = (1 / Comes Down)

Since Goes Up means inverse of Comes Down, hence

That means Goes Up = (1 / Comes Down)

From Eq2, we can say

Comes Down = (1 / Comes Down)

Which is Impossible!

Hence proved that this is wrong, and Whatever goes up must keep going up!

Hanky Panky
  • 121
  • 4